the mathematical model y= 2/x is an example of what variation? Joint variation, inverse variation, indirect variation, linear variation, or direct variation?​

Answers

Answer 1

Answer:

inverse variation

Step-by-step explanation:

Thiis is the model for inverse variation.   As x increases, y decreases, and vice versa.

Typical example:;  y = 5/x


Related Questions

a country’s population in 1990 was 123 million in 2002 it was 128 million

Answers

Answer:

whats the question

Step-by-step explanation:

Given 5 straight lines and 4 triangles in a plane. What is the maximum number of times that these lines
can cut the sides of these triangles?

Answers

I’m not sure but I’ll say 89

Receipt-of-goods discounts tend to be offered in cases where?​

Answers

The invoice is enclosed with the goods and arrives at the time that they do

\A and \B are supplementary angles. If m\A= (3x – 23) and m
\B = (2x – 12), then find the measure of \B.

Answers

2 supplementary angles when added together need to equal 180 degrees.

Add:

3x-23 + 2x-12 = 180

Simplify:

5x -35 = 180

Add 35 to both sides:

5x = 215

Divide both sides by 5:

X = 43

Now replace x with 43 in the equation for angle b:

2(43) -12 = 86-12 = 74 degrees

Angle B = 74 degrees

A workbench for a motorcycle is in the shape of parallelogram IJKL. The motorcycle is
raised and lowered as the angles of the parallelogram change. Which statement is
correct?



ZJ and ZL are supplementary
ZJ and ZK are congruent.
65° K
ZJ and ZK are supplementary
ZI and ZL are congruent.

Answers

Answer:

J and K are supplementary

Step-by-step explanation:

Given

The attached image

Required

The true statement

From the attached image, it is hard to determine the values of angles I and L as there are no enough information to determine their values.

However, J and K can be worked easily.

From the image

[tex]J = 65^o[/tex] --- vertically opposite angles

And:

[tex]K + 65^o = 180^o[/tex] ---- angle on a straight line

Substitute: [tex]J = 65^o[/tex]

[tex]K + J = 180^o[/tex]

When two angles add up to 180, the angles are supplementary.

Hence, (c) is correct

Karen purchased 3 gallons of yellow paint and 4 gallons on blue paint from the hardware store. The total cost was $105. Yellow
paint and blue paint sell for the same price per gallon. Which THREE statements are correct?

Answers

Answer:

In order to answer this properly, I would need to see the choices. However, I can tell you that:

-Each gallon of paint cost $15

-Karen spent $45 on yellow paint

-Karen spent $60 on blue paint

Step-by-step explanation:

Karen bought a total of 7 cans of paint, all the same price.

105/7=15.

15*3=45 (yellow paint)

15*3=60 (blue paint)

Hope this helps.

Which of the quadratic functions has the narrowest graph?

y = 2x^2
y = –x^2
y = 1/8x^2
y = 1/6x^2

Answers

A quadratic function's graph being wide or narrow is determined or depended on a-term:

[tex] \large{y = a {x}^{2} + bx + c}[/tex]

If |a| has a lot of value, for example a = 2 or a = 100. The graph will get narrower if increasing the value of |a|. On the other hand, If |a| has small value, for example a = 1/2 or a = 1/10000. The graph would be wide.

Also it does not matter if a-term is negative or not since a-term being positive or negative determines if a parabola is upward or downward. Only |a| determines how narrow/wide the graph is.

From the question, it is clear that the parabola y = 2x^2 is the narrowest graph since it has the highest |a| value out of all choices.

Answer

y = 2x^2

If f(x) = 6x + 2 and g(x) = 4x - 5, find f(x) - g(x).
A. 2x - 7
B. 2x + 7
C. 10x - 3
D. 10x + 7

Answers

D would be the answer

Answer: D

Step-by-step explanation:

set the functions to subtract

8 necklace have 864 beads in all how many beads will 15 such necklace have?​

Answers

864/8 gives you 108. There are 108 beads on each necklace. So for 15 necklaces, multiply 108 and 15. There are 1,620 beads in 15 necklaces.

Consider this as a simultaneous-move (static) game:
Player B
Left Right
Top 2, 2 3, 2
Player A
Bottom 1, 3 1, 4
1a) Write down the Best Response Correspondence for each of the two players.
1b) Does any player have a dominant strategy in this game? Explain.
1c) Find all Nash Equilibria in pure strategies of this game.
1d) Is there any Nash Equilibria in mixed strategies?

Answers

Answer:

Follows are the solution to the given points:

Step-by-step explanation:

For point a:

When Player A selects Top, Player B selects Left or Right.

Player B selects Right when player A selects Bottom

Thus, player B's best statement is correct.

When player B selects the left, then game A selects the top

When player B selects the right, player A selects the top

Hence, Player A's right response is Top.

For point b:

The main strategy inside the game is Player A, which would be Top. Since he won't choose Below except under situations since the payment to Bottom is below his payoff for Top irrespective of player B.

For point c:

Player A will purely pick Top & Player B right and player A also will pick Top and player B is right. Game A will's pattern. In this case, neither player has the motive to move away. There are two Nash balances since player B is paid the same amount, irrespective of what he's playing any strategy, as player A is always at the top.

For point d:

Consider if player B plays left with "q" probability and right with "1-q" probability. We're done

[tex]2q + 2(1-q) = 3q+4(1-q) \\\\2(1-q) - 4(1-q) = 3q - 2q \\\\-2(1 - q) = q \\\\-2 + 2q = q \\\\2q-q= 2 \\\\q= 2\\[/tex]

It is not possible since q is a chance that really can exceed 1. Hence, for this game, there is no mixed strategy nash balancing.

Solve the equation 2x^2 + 3 – 41 = –15 to the nearest tenth.

Hellllpppp

Answers

9514 1404 393

Answer:

 x = {-4.4, +2.9}

Step-by-step explanation:

We assume you want to solve ...

  2x^2 +3x -41 = -15

Adding 41 and factoring out the leading coefficient gives ...

  2(x^2 +3/2x) = 26

Dividing by 2 makes it ...

  x^2 +3/2x = 13

We can add the square of half the x-coefficient to "complete the square."

  x^2 +3/2x +(3/4)^2 = 13 +(3/4)^2

  (x +3/4)^2 = 13.5625 . . . . write the left side as a square

  x +3/4 = ±√13.5625 . . . . . take the square root

  x = -0.75 ±3.683 = {-4.433, +2.933} . . . . subtract 3/4 and evaluate

The solutions are approximately x = -4.4 and x = 2.9.

Phân biệt giá cấp 2 được áp dụng đối với

Answers

Second-degree price discrimination occurs when a company charges a different price for different quantities consumed, such as quantity discounts on bulk purchases.

mark me brainliestt :))

Can I get some help with my homework

Answers

Answer:

1) 37

2) 23

3) 4

4) 58

5) 67

6) 7

7) x = 14

  BC = 27

  CD = 61

  BD = 88

Step-by-step explanation:

1) Add them

2) Subtract them

3) Add them and set equal to 36

6x + 1 + x + 7 = 36

4) Add them and set equal to 9x - 39

47 + 3x + 10 = 9x - 39

Substitute x into 3x + 10 to find EF

5) Add them and set equal to 6x - 35

19 + 4x - 20 = 6x - 35

Substitute x into 6x - 35 to find UW

6) Add them and set equal to 7x-27

3x - 5 + x - 1 = 7x - 27

7) Add them. Solve for x and substitute.

4x - 29 + 5x - 9 = 7x - 10

C. Directions: Complete the table ( Area of a Circle).

Radius Diameter Area
16. _____ 7.5cm _______
17. 10cm _______ _______
18. 21cm ________ _______

pasagot po plss bukas na po pasahan ko​

Answers

Answer:

3.75cm 44.16cm²

20cm 314cm²

42cm 1384.74cm²

Step-by-step explanation:

Area of a circle = πr²

Where : = π = pi = 3.14

R = radius

the diameter is the straight line that passes through the centre of a circle and touches the two edges of the circle.

A radius is half of the diameter

16. Radius = 7.5 / 2 = 3.75

Area = 3.14 x 3.75² = 44.16

17. Diameter = 10 x 2 = 20

Area = 10² x 3.14 = 314

18. Diameter = 21 x 2 = 42

Area = 21² x 3.14 = 1384.74

Suppose the weights of apples are normally distributed with a mean of 85 grams and a standard deviation of 8 grams. The weights of oranges are also normally distributed with a mean of 131 grams and a standard deviation of 20 grams. Amy has an apple that weighs 90 grams and an orange that weighs 155 grams.

Required:
a. Find the probability a randomly chosen apple exceeds 100 g in weight.
b. What weight do 80% of the apples exceed?

Answers

Answer:

a) 0.0304 = 3.04% probability a randomly chosen apple exceeds 100 g in weight.

b) The weight that 80% of the apples exceed is of 78.28g.

Step-by-step explanation:

Normal Probability Distribution:

Problems of normal distributions can be solved using the z-score formula.

In a set with mean [tex]\mu[/tex] and standard deviation [tex]\sigma[/tex], the z-score of a measure X is given by:

[tex]Z = \frac{X - \mu}{\sigma}[/tex]

The Z-score measures how many standard deviations the measure is from the mean. After finding the Z-score, we look at the z-score table and find the p-value associated with this z-score. This p-value is the probability that the value of the measure is smaller than X, that is, the percentile of X. Subtracting 1 by the p-value, we get the probability that the value of the measure is greater than X.

Weights of apples are normally distributed with a mean of 85 grams and a standard deviation of 8 grams.

This means that [tex]\mu = 85, \sigma = 8[/tex]

a. Find the probability a randomly chosen apple exceeds 100 g in weight.

This is 1 subtracted by the p-value of Z when X = 100. So

[tex]Z = \frac{X - \mu}{\sigma}[/tex]

[tex]Z = \frac{100 - 85}{8}[/tex]

[tex]Z = 1.875[/tex]

[tex]Z = 1.875[/tex] has a p-value of 0.9697

1 - 0.9696 = 0.0304

0.0304 = 3.04% probability a randomly chosen apple exceeds 100 g in weight.

b. What weight do 80% of the apples exceed?

This is the 100 - 80 = 20th percentile, which is X when Z has a p-value of 0.2, so X when Z = -0.84.

[tex]Z = \frac{X - \mu}{\sigma}[/tex]

[tex]-0.84 = \frac{X- 85}{8}[/tex]

[tex]X - 85 = -0.84*8[/tex]

[tex]X = 78.28[/tex]

The weight that 80% of the apples exceed is of 78.28g.

Point A has coordinates (-24, -54)
Point B has coordinates (40, -46)
Find the equation of the perpendicular bisector of line AB.
ANSWER ASAP

Answers

Answer:

[tex]y=-8x+14[/tex]

Step-by-step explanation:

Hi there!

What we need to know:

A perpendicular bisector of a line segment is 1) perpendicular to the line segment and 2) passes through the midpoint of the line segmentPerpendicular lines always have slopes that are negative reciprocals (ex. -2 and 1/2)Linear equations are typically organized in slope-intercept form: [tex]y=mx+b[/tex] where m is the slope and b is the y-intercept (the value of  when x is 0)

1) Determine the midpoint of the line segment

Midpoint: [tex](\frac{x_1+x_2}{2} ,\frac{y_1+y_2}{2} )[/tex] where the coordinates of the endpoints are [tex](x_1,y_1)[/tex] and [tex](x_2,y_2)[/tex]

Plug in the endpoints (-24, -54) and (40, -46)

[tex](\frac{-24+40}{2} ,\frac{-54+(-46)}{2} )\\(\frac{-24+40}{2} ,\frac{-54-46}{2} )\\(\frac{16}{2} ,\frac{-100}{2} )\\(8 ,-50)[/tex]

Therefore, the midpoint of line AB is (8,-50).

2) Determine the slope of the line segment

This will help us find the equation of the perpendicular bisector.

slope = [tex]\frac{y_2-y_1}{x_2-x_1}[/tex] where two given points are [tex](x_1,y_1)[/tex] and [tex](x_2,y_2)[/tex]

Plug in the endpoints (-24, -54) and (40, -46)

[tex]= \frac{-46-(-54)}{40-(-24)}\\= \frac{-46+54}{40+24}\\= \frac{8}{64}\\= \frac{1}{8}[/tex]

Therefore, the slope of line AB is [tex]\frac{1}{8}[/tex].

3) Determine the slope of the perpendicular bisector

Because perpendicular lines always have slopes that are negative reciprocals, the slope of the perpendicular bisector is -8 (the negative reciprocal of 1/8). Plug this slope into [tex]y=mx+b[/tex]:

[tex]y=-8x+b[/tex]

4) Determine the y-intercept (b) of the perpendicular bisector

[tex]y=-8x+b[/tex]

Recall that we found the midpoint of line AB, (8,-50). The perpendicular bisector passes through this point. Plug (8,-50) into [tex]y=-8x+b[/tex] and solve for b:

[tex]-50=-8(8)+b\\-50=-64+b[/tex]

Add 64 to both sides to isolate b

[tex]-50+64=-64+b+64\\14=b[/tex]

Therefore, the y-intercept of the line is 14. Plug this back into [tex]y=-8x+b[/tex]:

[tex]y=-8x+14[/tex]

I hope this helps!

PLS HELP! I NEED TO FIND THE SURFACE AREA OF THIS CYLINDER!

PLS PROVIDE A STEP BY STEP EXPLANATION! ❤️

Answers

Exact Surface Area = 378pi

Approximate Surface Area = 1186.92

The approximate surface area uses pi = 3.14

The units are cm^2 or "square cm".

=========================================================

Work Shown:

SA = surface area of cylinder

SA = 2*pi*r^2 + 2*pi*r*h

SA = 2*pi*7^2 + 2*pi*7*20

SA = 2*pi*49 + 2*pi*140

SA = 2*49*pi + 2*140*pi

SA = 98pi + 280pi

SA = 378pi ..... exact surface area

SA = 378*3.14

SA = 1186.92 ..... approximate surface area

----------

Side note: The diameter 14 cuts in half to get the radius r = 7

Answer:

1187.52

Step-by-step explanation:

Use the formula [tex]2\pi rh+2\pi r^{2}[/tex].

r= 7 (half of your diameter, which is 14)

h = 20

Fill in your radius and height.

[tex]2\pi (7)(20)+2\pi (7^{2})[/tex]

Enter your equation into a calculator and you'll get 1187.52202.

When rounded to the nearest hundredths, you get 1187.52

Consider these functions:
Rx) = 5x^2 + 2
g(x) = x^2 - 1
What is the value of g(f(1))?
A.
2.
OB.
oa
C.
22
D.
48

Answers

Answer:

D

Step-by-step explanation:

F(1)=7

G(f(1)= g(7)= 49-1=48

What is the volume of sphere with radius 13 ft?

Answers

Answer:

[tex]\displaystyle V = \frac{8788 \pi}{3} \ ft^3[/tex]

General Formulas and Concepts:

Pre-Algebra

Order of Operations: BPEMDAS

Brackets Parenthesis Exponents Multiplication Division Addition Subtraction Left to Right

Geometry

Volume of a Sphere Formula: [tex]\displaystyle V = \frac{4 \pi}{3}r^3[/tex]

r is radius

Step-by-step explanation:

Step 1: Define

Identify variables

r = 13 ft

Step 2: Find Volume

Substitute in variables [Volume of a Sphere Formula]:                                 [tex]\displaystyle V = \frac{4 \pi}{3}(13 \ ft)^3[/tex]Evaluate exponents:                                                                                         [tex]\displaystyle V = \frac{4 \pi}{3}(2197 \ ft^3)[/tex]Multiply:                                                                                                             [tex]\displaystyle V = \frac{8788 \pi}{3} \ ft^3[/tex]

Answer:

The volume of this sphere is equal to [tex]2929\frac{1}{3} \pi ft^{3}[/tex]

Step-by-step explanation:

In order to solve this question, we need to know the formula for the volume of a sphere which is...

[tex]V = \frac{4}{3}\pi r^{3}[/tex]  ("V" is the volume of the sphere, and "r" is the radius of the sphere)

Now we have to substitute the values that we already know into the formula, and we will get that...

[tex]V = \frac{4}{3}\pi r^{3}\\\\V = \frac{4}{3} \pi (13ft)^{3} \\\\V = \frac{4}{3} \pi (2,197ft^{3} )\\\\V = 2,929\frac{1}{3} \pi ft^{3}[/tex]

Therefore, the volume of this sphere is equal to [tex]2929\frac{1}{3} \pi ft^{3}[/tex]

Darcy gave her hairstylist a $ 4.90 The tip was 14​% of the cost of the haircut . Write an equation to find​ b, the cost of the haircut.

Answers

Answer:

Equation: 4.90/b = 14/100

Solution: b = $35

Step-by-step explanation:

Variable b = cost of the haircut

Solve for b:

4.90/b = 14/100

490 = 14b

35 = b

Check your work:

35 × 0.14 = 4.90

Correct!

In a game, there are 12 identical balls of which seven are red and five are green.

Five red balls and two green balls have number ‘2’ written on them. The rest of the

red balls have number ‘1’ written on them, and the rest of the green balls have the

number ‘3’ written on them. A random sample of three balls is selected without

replacement. Let denotes the event that all the balls selected are red and

denotes that the sum of numbers of the three balls is equal to 6. Calculate:

(i) P(A) ,

(ii) P(B),

(iii)P ( A∩ B),

(iv)P(A|B).​

Answers

Answer:

its number 2 and if its a mutable answers writ 3 also

The probabilities are: (i) P(A) = 1/6

(ii) P(B) = 38/55

(iii) P(A ∩ B) = 1/110

(iv) P(A|B) ≈ 0.00152

To calculate the probabilities, let's first find the total number of ways to choose 3 balls out of the 12 balls.

Total number of ways to choose 3 balls out of 12 = 12C3 = (12 * 11 * 10) / (3 * 2 * 1) = 220

(i) P(A): Probability that all three balls selected are red.

Number of ways to choose 3 red balls out of 7 red balls = 7C3 = (7 * 6 * 5) / (3 * 2 * 1) = 35

P(A) = Number of favorable outcomes / Total number of outcomes = 35 / 220 = 1/6

(ii) P(B): Probability that the sum of the numbers on the three balls is equal to 6.

The possible combinations that sum up to 6 are: (2, 2, 2), (2, 2, 1), and (1, 1, 3).

Number of ways to choose 3 balls such that their sum is 6:

- For (2, 2, 2), we have 1 choice for each color, so 1 * 1 * 1 = 1 way.

- For (2, 2, 1), we have 1 choice for each color, so 1 * 1 * 1 = 1 way.

- For (1, 1, 3), we have 6 choices for the first red ball (all are labeled '1'), 5 choices for the second red ball (since one '1' is already taken), and 5 choices for the green ball labeled '3', so 6 * 5 * 5 = 150 ways.

Total number of ways to choose 3 balls with sum 6 = 1 + 1 + 150 = 152

P(B) = Number of favorable outcomes / Total number of outcomes = 152 / 220 = 38/55

(iii) P(A ∩ B): Probability that all three balls selected are red and the sum of their numbers is equal to 6.

From the above calculations, we know that there are 1 way to choose (2, 2, 2) and 1 way to choose (2, 2, 1) such that all three balls are red and the sum is 6.

P(A ∩ B) = Number of favorable outcomes / Total number of outcomes = 2 / 220 = 1/110

(iv) P(A|B): Probability that all three balls selected are red, given that the sum of their numbers is equal to 6.

P(A|B) = P(A ∩ B) / P(B) = (1/110) / (38/55) = (1/110) * (55/38) ≈ 0.00152 (rounded to five decimal places).

To know more about probabilities:

https://brainly.com/question/29381779

#SPJ2

3x-1= 7x+5
A: -0.6 C: 1.5
B: -1.5 D: 0.6

Answers

Answer:

x = -1.5

Step-by-step explanation:

3x-1= 7x+5

Subtract 3x from each side

3x-3x-1= 7x-3x+5

-1 = 4x+5

Subtract 5 from each side

-1-5 = 4x+5-5

-6 = 4x

Divide by 4

-6/4 = 4x/4

-3/2 =x

x = -1.5

The answer is = -1.5

At the start of a month, Sasha and Natalia each have a certain amount of money.
Sasha has $400 and saves $20 each week. The graph below shows the amount of money in Natalia's account each week
Whose monthly activity shows a greater rate of change, and by how much?
A) Sasha, by $10/week
B)Sasha, by $19/week
C) Natalia, by $10/week
D) Natalia, by $19/week

Answers

Answer:

Option (A)

Step-by-step explanation:

Sasha has an amount of $400 and saves $20 per week.

If we graph the savings of Sasha, her savings per week will be defined by the slope of the line = $20 per week

Similarly, from the graph attached,

Slope of the line given in the graph = Per week savings of Natalia

Slope of line passing through (0, 190) and (2, 210) will be,

Slope = [tex]\frac{y_2-y_1}{x_2-x_1}[/tex]

          = [tex]\frac{210-190}{2-0}[/tex]

          = 10

Therefore, per week savings of Natalia = $10

Difference in savings of Sasha and Natalia = 20 - 10 = $10 per week

Here, Sasha shows the greater rate of change by $10 per week

Therefore, Option (A) will be the answer.

if 3x+15=93 what is the value of X​

Answers

Answer:

26=x

Step-by-step explanation:

-15 from both sides

3x=78 /:3

x=26

Due to the heavy rains, all pallets of blocks have become wet and the weight of each block has altered. It is desired to estimate the average weight of the blocks thus wet. For this, a sample of 15 blocks is selected and the following weights are recorded in kilograms: 45.8, 43.9, 46.2, 48.5, 47.2, 45.8, 49.1, 43.9, 47.6, 42.8, 46.5, 44.9, 46.8, 45.8 and 44.6; Estimate µ, the average weight of all pallets of blocks, and determine a bound for the estimation error.

Answers

Answer:

To find the estimated value we add all of them together then divide it by the amount of them

45.8+43.9+46.2+48.5+47.2+45.8+49.1+43.9+47.6+42.8+46.5+44.9+46.8+45.8 +44.6=689.4

now we divide it by 15 since there is 15 blocks so

689.4 divided by 15=45.96

so the average weight is 45.96

In a class of 30 pupils the ratio of left-handed pupils to right-handed pupils is 1:9.
How many pupils are right-handed?

Answers

Step-by-step explanation:

Pupils are right-handed: 30:10x9=27

2 divide by 7/6
pls help

Answers

Answer:

7/6 ÷ 2 = 7/12 in fraction form. 

7/6 ÷ 2 = 0.5833 in decimal form

Step-by-step explanation:

6 divided by 7 equals 6/7 or 0.86

B.
Classity the following polygons as to regular or irregular
polygon. Write your answer on your answer sheet
1
2
3
4
5 ​

Answers

Answer:

If I'm correct I think 4 is the polygon

Someone help me on this percent question

Answers

Answer: 6%

Hope it helps :)

Step-by-step explanation:

Price of mixer = 45.00$

Price of mixer with tax = 47.70

Tax added to the price of mixer = 45 - 47.70

                                                      = 2.70$

Percentage of Tax added to the price of mixer

 =  2.70 = P multiplied by 45

  (divide 2.70 by 45 and 45 by 45 as well)                                  

 = 0.06 = P

  (we have to now convert the decimal to a percentage)

= 0.06 multiplied by 100 = P

   6% = P

a garden pond is in the shape of a rectangle that measures 5 m by 3 m. a stone path is built all around the pond. this path is the same width all the way around. the area of the pond and the path together is 39 m². how wide is the path?​

Answers

Answer:

1.16 m (approximately)

Step-by-step explanation:

Let x be the width of the path.

Total area  = (5+2x)(3+2x) = 39

Expand and simplify

4x²+ 16x+15-39 = 0

4x² + 16x - 24 = 0

Simplify

x² + 4x -6 = 0

Rational factoring does not work, so use quadratic formula

x = +/- sqrt(10) -2

= 1.16 or -5.16 (reject)

= 1.16 m (approximately)

Other Questions
write the reciprocal [tex] \frac{3}{8 } = [/tex] I need help with this question What is the colour of Copper sulphate solution discuss three contributing factors that led to the xenophobia The overwhelmingly supportive response on behalf of many Oklahomans that came out of the Oklahoma City bombing tragedy became known as the "Oklahoma __________." A. Outpouring B. Vibe C. Nature D. Standard Which sequence is geometric? The question is in the screenshot Microwaves have lowfrequencies. What does thistell you about thewavelengths of microwaves?A. They have long wavelengths.B. They have short wavelengths.C. Frequency is unrelated to wavelength. Un campo magnetico uniforme B, con magnitud 1.2 mT, apunta verticalmente hacia arriba a lo largo del volumen del salon en que usted esta sentado. Un proton de 5.3 MeV se mueve horizontalmente de sur a norte a traves de cierto punto en el salon. Que fuerza magnetica deflectora actua sobre el proton cuando pasa por este punto? Cual es su aceleracion, la masa del proton es de 1.67x1027 kg. Resp. 6.1x1015 N; 3.7x1012 m/s2. If f(x) = 3x - 1 and g(x) = x + 2, find (f + g)(x).O A. 2x-3O B. 4x+1. 3 3O D. 2x-1 Susie leaves to go shopping driving 45 miles per hour toward the Happy Shopper Grocery Store. Susie is originally 20 miles due west of the store. Sammie leaves for the Happy Shopper Grocery Store at the same time driving 50 miles per hour on the same road but she is 25 miles due EastIf they keep driving and do not stop at the store, how long (in hours) will it take for them to pass on the road? Where relative to the store is this (please state a distance and whether they are East or West of the store). Round the time (in hours) to the nearest thousandth and the distance to the nearest tenth if needed. A sample of 46 observations is selected from one population with a population standard deviationof 4.1. The sample mean is 102.0. A sample of 48 observations is selected from a secondpopulation with a population standard deviation of 5.8. The sample mean is 100.1. Using the 0.05significance level, is there a difference between the two samples? Conjugate the verb found in parenthesis.Angie __________ al mercado (it).A: vanB: vasC: vaD: vamos Why would the following statement not be a good way to end a phone conversation with someone?"Hey, I've gotta go. My TV show's back on."Answer: Because there is no promise of future interaction. The dean of the UTC Engineering School at a small Florida college wishes to determine whether the grade-point average (GPA) of a graduating student can be used to predict the graduate's starting salary. More specifically, the dean wants to know whether higher GPAs lead to higher starting salaries. Records for 23 of last year's Engineering School graduates are selected at random, and data on GPA and starting salary ( in $thousands) for each graduate were used to fit the model The dependent variable is____________________________. reflection across the y-axis Llustrate the five (05) stages of Weitzel and Jonsson's Model of Organizational decline by providing real-world examples Differentiate between a derived quantity and a derived unit. can someone please help PLEASE !!